Abhyaas Prelims 2024 Csat Test 1 English

You might also like

Download as pdf or txt
Download as pdf or txt
You are on page 1of 46

VISIONIAS

www.visionias.in

Test Booklet Series

TEST BOOKLET

CSAT ABHYAAS TEST – 1 (5058) – 2024


C
Time Allowed: Two Hours Maximum Marks: 200

INSTRUCTIONS
1. IMMEDIATELY AFTER THE COMMENCEMENT OF THE EXAMINATION, YOU SHOULD CHECK THAT THIS
BOOKLET DOES NOT HAVE ANY UNPRINTED OR TORN OR MISSING PAGES OR ITEMS ETC. IF SO, GET IT
REPLACED BY A COMPLETE TEST BOOKLET.

2. ENCODE CLEARLY THE TEST BOOKLET SERIES A, B, C OR D AS THE CASE MAY BE IN THE APPROPRIATE PLACE
IN THE ANSWER SHEET.

3. You have to enter your Roll Number on the Test Booklet in


the Box provided alongside. DO NOT write anything else on
the Test Booklet.

4. This Test Booklet contains 80 items (Questions). Each item is printed in English. Each item comprises four
responses (answers). You will select the response which you want to mark on the Answer Sheet. In case you
feel that there is more than one correct response, mark the response which you consider most appropriate.
In any case, choose ONLY ONE response for each item.

5. You have to mark all your responses ONLY on the separate Answer Sheet provided. See direction in the
answers sheet.

6. All items carry equal marks. Attempt all items. Your total marks will depend only on the number of correct
responses marked by you in the answer sheet. For every incorrect response one-third of the allotted Marks
will be deducted.

7. Before you proceed to mark in the Answer sheet the response to various items in the Test booklet, you have
to fill in some particulars in the answer sheets as per the instruction sent to you with your Admission
Certificate.

8. After you have completed filling in all responses on the answer sheet and the examination has concluded,
you should hand over to Invigilator only the answer sheet. You are permitted to take away with you the Test
Booklet.

9. Sheets for rough work are appended in the Test Booklet at the end.

DO NOT OPEN THIS BOOKLET UNTIL YOU ARE ASKED TO DO SO


1 www.visionias.in ©Vision IAS

Telegram: UPSC_SARTHI
1. Number 1X62 is added to 58Y9 and the sum 2. Based on the above passage, the following

obtained is Z401, where X, Y and Z are single- assumptions have been made:

digit odd prime integers, then the only possible 1. The Mughal administration, by making
villages the units of assessment, aimed to
value of X + Z is:
foster a sense of self-governance and
(a) 12
empower local communities.
(b) 10
2. The Mughal administration endeavoured
(c) 7 to optimise the accumulation of its
(d) 8 financial resources.
Which of the assumptions given above is/are

Directions for the following 3 (three) items: correct?

Read the following three passages and answer the (a) 1 only
(b) 2 only
items that follow each passage. Your answers to these
(c) Both 1 and 2
items should be based on the passages only.
(d) Neither 1 nor 2
Passage – 1

The simplest form of land revenue was crop sharing. Passage – 2


From this simple sharing, we can trace the evolution of The idea of sustainable development has been there,

other systems of assessment, which were designed to ever since human beings started using natural

reduce the burden of work and expenses for the resources for their development. All forms of
development, be it economic, educational,
Mughal administration. One such assessment method
technological, agricultural, or social, have taken a toll
was Kankut (Kan = grain, Kut = estimate). According
on the consumption of environmental resources. The
to one of the leading historians, one major aim of the
declaration of the United Nations Conference on
Mughal administration was to take away the bulk of
Human Environment recognized the finite nature of
the peasant’s surplus. The shares of the crop taken
resources available on earth and emphasized their
under the ‘batai’ or Kankut varied with crops and judicious use to remain available for future
localities. In Northern India, the unit of assessment generations. In 1983, the UN General Assembly

was the village. The revenue due from a village was established a commission, which later came to be

fixed regarding its productive capacity usually for the known as the World Commission on Environment and
Development (WCED), to come up with a report on
year. However, this sum was not distributed by
the state of the environment and propose sustainable
assessors over the individual peasants.
development to the year 2000 and beyond.

www.visionias.in 2 ©Vision IAS

Telegram: UPSC_SARTHI
3. Which of the following statements best 4. Based on the above passage, the following

reflects the most logical, crucial, and rational assumptions have been made:
1. The detrimental effects of human activity
message conveyed by the passage?
on the environment are interconnected and
(a) Sustainability necessitates halting all
cascading in nature.
development works.
2. The pernicious consequences of human
(b) The imperative of sustainable actions on the environment are irreparable
development arises from the inherent and cannot be reversed.

constraints imposed by the finite nature of 3. Being the root cause of environmental

Earth's resources. degradation, there is a need to reduce the


urban population only.
(c) The genesis of sustainable development
Which of the assumptions given above is/are
can be traced back to the efforts of United
incorrect?
Nations and WCED.
(a) 1 and 3 only
(d) None of the above (b) 2 and 3 only
(c) 1 only

Passage – 3 (d) All 1, 2 and 3

Human activities are causing rapid environmental


5. Subtracting 3 from a number and then
changes on the Earth. Humans have greatly impacted
multiplying this result by 4 is equivalent to
the environment including impacts on biophysical
multiplying the original number by 4 and then
environments, composition, diversity of terrestrial
subtracting:
ecosystems and other resources etc. Changes in (a) 5
environmental limiting factors caused by humans are (b) 3

likely to result in a major loss of plant diversity; plant (c) 12


(d) 8
communities dominated by weedier species can

change the ecological functioning. On our current path,


6. If p1 and p2 are prime numbers such that p1 <
ice cap melt will cause sea levels to rise to levels
p2 < 50 and p1 + p2 is a factor of 315, then how
where many major cities will be at very high risk of many values of p1 is/are possible?
flooding, and natural disasters will cause damage to (a) 1

our communities at catastrophic levels on a much more (b) 3

regular basis. All these problems are exacerbated by an (c) 5


(d) 7
ever-growing human population.
www.visionias.in 3 ©Vision IAS

Telegram: UPSC_SARTHI
7. In a certain code, GIVEN is written as Directions for the following 4 (four) items:
MDVJH. How would SHARK be written in Read the following four passages and answer the
that code? items that follow each passage. Your answers to these
(a) JSAIT items should be based on the passages only.
(b) JQAIT Passage – 1
(c) TIAQJ Democracy and Media are two major elements in the
process of development, as development is no longer
(d) JQIAT
viewed as the sole responsibility of the elected
governments. In this era of globalization, the media
8. Ajay intended to order 4 pairs of black socks
has emerged as one of the most powerful components
and some pairs of blue socks. The price of the
of social management. The role of mass media in
black socks per pair was twice that of blue.
shaping public opinion is well known. But from the
However, the order was incorrectly filled in days of the Capitalist revolution, the press was
such a manner that the number of pairs of the accorded the status of the fourth estate for no other
two colours got interchanged. This increased reason than its power to influence the minds and thus
the bill by 50%. Find the number of pairs of accelerate the political process in favour of bourgeois
blue socks in the original intended order. democracy. The role of media is twofold. On the one
(a) 15 hand, the media influences the policymakers by
(b) 1 putting forth the opinions expressed by various groups
(c) 12 including educationists, journalists and experts, leaders
(d) 16 of different political parties, religious leaders, workers,
peasants Unions, etc. On the other hand, the media
pressurizes the policy makers or the authorities to act
9. Six positive integers a, b, c, d, e, and f have an
in response to people's interests and demands by
average of 36. It’s also known that a < b < c <
opening a debate and educating the masses.
d < e < f. What could be the greatest ratio of d
to c?
11. Which of the following statements can be
(a) 65:2
inferred from the above passage?
(b) 15:1
1. The media serves as a conduit between
(c) 45:2
citizens and policy makers.
(d) 23:1 2. The role of the fourth estate is inherently
biased towards promoting bourgeois
10. Three different faces of a cube are coloured in democracy.
three different colours—black, green and blue. 3. The rise of media as a powerful
This cube is now cut into 216 smaller but component of social management comes
identical cubes. N are the number of cubes that at the cost of the democratic ethos.
are not coloured on any of their faces. Which Choose the correct answer using the codes
of the following best estimates the value of N? given below.
(a) 65 < N < 130 (a) 1 only
(b) 120 ≤ N ≤ 145 (b) 1 and 2 only
(c) 3 only
(c) 60 ≤ N ≤ 100
(d) 2 and 3 only
(d) 10 < N < 35
www.visionias.in 4 ©Vision IAS

Telegram: UPSC_SARTHI
Passage – 2 Passage – 3
The process of socialization begins at birth. It is a
In recent years there has been an enormous increase in
continuous process because social learning never ends.
the studies on social movement in India. The growth of However, childhood is the most important stage in the
interest is largely a result of the increasing number of process of socialization during which a child
internalizes or learns most of the values, beliefs,
movements surfacing in post-colonial India. The
norms, attitudes, and behaviour patterns of its family.
modern Indian working class arose because of the The parents can be viewed as the socializing agents. In
development and growth of factory industries in India some societies, the education and socialization of the
young take place without extensive formal educational
from the second half of the nineteenth century. It is institutions. We may also mention that all learning is
however about the turn of the twentieth century, it took not socialization since some of what one learns may
not be relevant or necessary for participation in given
the shape of the working class. The actions of the
social roles.
working class in the earliest stage were sporadic and

unorganized and hence were mostly ineffective. It was 13. Which of the following statements reflects the
most logical, rational, and crucial message
only from the late 19th century in Madras, and from
conveyed by the author of the passage?
the second decade of the twentieth century in Bombay (a) A child's formative years set the stage for
a lifetime of social learning and
that serious attempts were made for the formation of
behavioural development.
associations that could lead to organized forms of (b) All the knowledge we pick up throughout
protests. our lives contributes to the process of
socialization.
(c) A few societies do not need formal
12. Which of the following statements best education system at all.
reflects the most logical, rational, and crucial (d) Parents are the most important agents of
socialisation for their children.
message conveyed by the passage?

(a) All social movements in post-colonial Passage – 4


In any discussion or discourse on social change in
India are directly related to the working
Indian society, the impact of the British colonial rule
class. occupies an important place. The British colonial
(b) The success of social movements depends rulers, to bring about desired changes in the points of
view of the people, brought out a new system of
solely on their level of organization and
education. In the initial stages, British rule influenced
formal associations. the port and coastal cities. A new system of education
was introduced to achieve the goal of nurturing a class
(c) The episodic nature of early working-class
in India which would sustain British rule. To serve
actions contributed to their ineffectiveness their interests the British rule introduced a new
in achieving the desired outcomes. Western education, new means of transport and
communication, new technology and a new system of
(d) Study of social movements in India is
judiciary. These, in turn, released new forces of change
labour centric. and the Indian society could never be the same again.
www.visionias.in 5 ©Vision IAS

Telegram: UPSC_SARTHI
14. Based on the above passage, the following 17. Four persons - A, B, C and D are positioned at
assumptions have been made: the four corners of a piece of a plot as shown
1. The colonial education system played a in the following diagram.
catalyzing role in positive social
transformations within Indian society.
2. The British trained coastal India to create
human resources which can help in their
exports worldwide.
Which of the assumptions given above is/are
correct?
From their original positions, B and D move
(a) 1 only
one and a half length of the side in clockwise
(b) 2 only
direction. Which one of the following
(c) Both 1 and 2
statements must now be true?
(d) Neither 1 nor 2
(a) Both B and D are at the midpoints
between A and C.
15. Consider the following statements.
(b) B is at the midpoint between A and C, and
I. A is the brother of B.
D is at the corner occupied by A.
II. B is the brother of C.
III. C is the brother of D. (c) D is closer to A than he is to C and B is

IV. D is the sister of E. closer to C than he is to A.


(d) D is closer to C than he is to A and B is

Which of the following statements is not closer to A than he is to C.

necessarily true?
(a) D is the sister of C. 18. One man can do as much work in 2 days as a

(b) B and C are brothers. transgender can do in 3 days. A woman does

(c) A, B, C are males and D is a female. three- fourth the work in a day as a

(d) D and E are sisters. transgender. A contractor hires 76 pairs of


hands, men, transgenders and women in the

16. The following sequence is generated by ratio 8: 7: 4 and pays them in all Rs 8,800 at

dividing 55 by natural numbers: the end of the day’s work. What must the daily

0, 1, 1, 3, 0, X, 6, 7, ...... wage of a transgender be, if the wages are

What is X? proportional to the amount of work done?

(a) 3 (a) Rs 200

(b) 2 (b) Rs 100


(c) 1 (c) Rs 250
(d) 0 (d) Rs 150
www.visionias.in 6 ©Vision IAS

Telegram: UPSC_SARTHI
19. Three statements are followed by two 21. How many ordered triplets (x, y, z) satisfy the
conclusions numbered I and II. You have to equation (x + y + z) = 21, where x, y and z are

take the given statements to be true even if natural numbers divisible by 3?

they seem to be at variance with commonly (a) 12


(b) 15
known facts and judge the conclusions on their
(c) 9
basis.
(d) 10
Statements:
I. Some frooti are maaza
22. Birthday of a child X falls on Monday 1st
II. No maaza is slice
January 2020. Find out in which upcoming
III. All slice are fanta
year X will celebrate his birthday again on the
same day.
Conclusions: (a) 2048
I. Some frooti are definitely not slice (b) 2025
II. Some fanta are definitely not maaza. (c) 2026
(d) 2028
Which of the conclusions given above is/are
correct? Directions for the following 2 (two) items:

(a) Only conclusion I follows Read the following two passages and answer the items

(b) Both conclusions I and II follow that follow each passage. Your answers to these items
should be based on the passages only.
(c) Neither conclusion I nor II follows
Passage – 1
(d) Only conclusion II follows
Agriculture, with its allied sectors, is the largest source
of livelihood in India. 70 per cent of its rural
20. In a party, out of total guests, 60% are women.
households still depend primarily on agriculture for
If 80% of the guests are married couple (i.e.
their livelihood. While achieving food sufficiency in
form a pair of husband-wife) and rest are
production, India still accounts for a quarter of the
unmarried, then which of the following world’s hungry people and is home to over 190 million
statement(s) is/are correct? undernourished people. While agriculture in India has
1. All men are married. achieved grain self-sufficiency, the production is,
2. More than 30% women are unmarried. resource-intensive, cereal-centric and regionally
Select the correct answer using the codes biased. The resource-intensive ways of Indian
given below: agriculture have raised serious sustainability issues

(a) 1 only too. The increased feminisation of agriculture is

(b) 2 only mainly due to increasing rural-urban migration by


men, the rise of women-headed households and growth
(c) Both 1 and 2
in the production of labour-intensive cash crops.
(d) Neither 1 nor 2
www.visionias.in 7 ©Vision IAS

Telegram: UPSC_SARTHI
23. Based on the above passage, the following 24. Which of the following statements can be
inferred from the above passage?
assumptions have been made:
1. Contemporary Indian society, despite
1. India suffers from a paradox of food experiencing major transformations, still
upholds some traditional gender roles.
sufficiency and persistent hunger existing
2. Indian society is lagging in gender
simultaneously. equality compared to other societies
2. In Indian context, feminisation of undergoing similar transformations.
Choose the correct answer using the codes
agriculture will not lead to women given below.
empowerment. (a) 1 only
(b) 2 only
Which of the assumptions given above is/are (c) Both 1 and 2
correct? (d) Neither 1 nor 2

(a) 1 only
25. What is the sum of the first 50 positive even
(b) 2 only integers?
(a) 2365
(c) Both 1 and 2
(b) 2450
(d) Neither 1 nor 2 (c) 2550
(d) 2750

Passage – 2 26. Six Investors - Kavita, Laxman, Manish, Niraj,


Contemporary Indian society has been exposed to the Om and Parvati, purchased different number
of shares of HUL Company. No two persons
broad processes of social transformation, agricultural purchased the same number of shares. Further,
modernization and economic development, it’s known that:
1. Three people bought more shares that
urbanization, rapid industrialization, and globalization.
Kavita but less than Om.
It is not enough to say that any society consists of men 2. Parvati, who purchased 37 shares,
purchased less shares than Niraj.
and women. It is equally important to look at how the
3. Manish purchased 34 shares, which is less
two groups of people interact, as well as at the roles than what Laxman bought.
4. When they are arranged in ascending order
and expectations each group has of the other. The first
of the number of shares bought, Manish
idea on gender role differences which a child acquires and Laxman are adjacent to each other.
5. Om did not purchase the maximum
is that of women of one’s family marrying and leaving
number of shares.
their homes to live with different groups of people. How many statements given above are
Not only is women’s productive work within the house required to determine who purchased the
maximum number of shares?
unpaid, but also it often is not understood how the (a) Only two
multiplicity of roles may result in conflict in their (b) All five
(c) Only three
performance. (d) Only four.
www.visionias.in 8 ©Vision IAS

Telegram: UPSC_SARTHI
27. On a typical day, a restaurant sells n grilled 29. If the cost of living in a certain city increased
cheese sandwiches for p dollars each. Today, by 20% in the 10 years from 1980 to 1990, and
however, the manager reduced the price of
increased by 50% in the 20 years from 1980 to
grilled cheese sandwiches by 30% and as a
2000, what was the percent increase in the cost
result sold 50% more of them than usual.
Which of the following represents the revenue of living from 1990 to 2000?

for today’s grilled cheese sandwich sales? (a) 15%


(a) 0.5n - p - 0.3 (b) 20%
(b) 1.05np
(c) 25%
(c) 1.20np
(d) 30%
(d) 1.50np

28. Consider the following Question and the 30. Consider the following statements and
Statements:
conclusions:
Question: There are 5 members Abhinav,
Statements:
Bhavna, Chanda, Deepu, and Erapalli in a
If A, B, C and D are whole numbers such that:
family. What is the relation of Erapalli with
Bhavna? 1. B + C + D = 146

2. C + D + A = 198
Statement-1: Abhinav and Bhavna are a
3. A + B + C = 128
married couple.
4. D + A + B = 164
Statement-2: Deepu is the father of Chanda.
Statement-3: Erapalli is Deepu's son.
Statement-4: Abhinav and Chanda are sisters. Conclusions:

I. B is the smallest number and B = 14.


Which one of the following is correct in
II. D is the largest number and D = 86.
respect of the above Question and Statements?
III. A is the largest number or A = 66.
(a) Statement-1, Statement-2 and Statement-3
are sufficient to answer the Question.
(b) Statement-1, Statement-3 and Statement-4 Which of the above conclusion(s) is/are true?
are sufficient to answer the Question. (a) Only l
(c) All four statement together are sufficient
(b) Only ll
to answer the Question.
(c) Both l and lll
(d) All four statements together are not
sufficient to answer the Question. (d) Only lll

www.visionias.in 9 ©Vision IAS

Telegram: UPSC_SARTHI
Directions for the following 4 (four) items: Passage – 2
Read the following three passages and answer the The country's longest sea bridge, the 22-km long six-
items that follow each passage. Your answers to these lane Atal Setu was inaugurated recently. Formally
items should be based on the passages only.
called the Atal Bihari Vajpayee Sewari-Nhava Sheva
Passage – 1
Atal Setu or the Mumbai Trans Harbour Link, the
COP28’s declaration asking countries to “transition
away” from fossil fuels is certainly historic. But that bridge has been built at a cost of Rs 17,840 crore. By
only indicates how little UNFCCC meets have using it, commuters will be able to cut the travel time
achieved since the Paris Pact of 2015. Two years ago, between central Mumbai and the fast-growing Navi
at Glasgow, delegates had issued a call to “phase down Mumbai from 2 hours to about 20 minutes. But
out coal” but the issue remained contentious and
viewing the Atal Setu merely as a way to decongest
divisions amongst nations were even sharper on
the traffic woes in India's financial capital would be
eliminating all fossil fuels. In the run-up to COP28, it
was apparent that putting the matter on the back burner akin to missing the forest for the trees. It is indeed
had pushed the world perilously close to the threshold another key step in India's bid to transform not only its
beyond which climate change becomes irreversible. physical infrastructure but also its global image as an
Even then, Dubai showcased the difficulty of framing investment destination. Focus on boosting
a language on fossil fuel elimination, acceptable to all.
infrastructure — be it physical or digital — is of
After working overtime, delegates agreed on a
paramount importance if India wants to truly take
declaration that calls for a “transitioning away from
advantage of the disenchantment with China that's
fossil fuels in energy systems, in a just, orderly and
equitable manner”. It addresses the concerns of India, setting in several parts of the world. Developed
China, and other emerging economies. However, the country markets and investors are looking for countries
failure to secure explicit commitments and set targets that can replace China in the global supply chain. India
seems to have disappointed small island states and can leverage this opportunity to become a developed
other countries most vulnerable to climate change. The
country in the next quarter of a century.
declaration merely talks of “accelerating action in this
critical decade, to achieve net zero by 2050 in keeping
with the science”. This is, at best, an incremental 32. Which one of the following statements best
advance. reflects the crux of the passage?
(a) Atal Setu reflects the peak of the
31. Which one of the following statements best development of physical infrastructure in
reflects what is implied by the passage?
India.
(a) Paris Pact and the pacts thereafter gave a
(b) The advantages of building Atal Setu go
big boost to climate change mitigation
far beyond mere physical infrastructure
efforts, globally.
(b) All the countries have unanimously agreed development.
on the elimination of fossil fuels. (c) Atal Setu will help decongest the traffic
(c) There is a lack of consensus on the woes in Mumbai.
concrete steps required for a robust (d) Many forests have been cut to construct
climate action framework.
Atal Setu.
(d) Climate change is irreversible now.
www.visionias.in 10 ©Vision IAS

Telegram: UPSC_SARTHI
Passage – 3 34. With reference to the passage, the possible
Women’s empowerment is a prerequisite for achieving advantages of using drone technology in
Viksit Bharat or developed India. The NAMO Drone agriculture include:
1. Efficient application of pesticides and
Didi Scheme provides new work opportunities to
fertilizers
women. It makes them the backbone of the rural
2. Making agriculture completely self-
economy. This will go a long way in making our
sufficient
agrarian family culture more equitable and stronger. A 3. Increased agricultural productivity
woman drone pilot can play a vital role in increasing 4. Time saving
agricultural productivity and efficiency. The 5. Reduction in hardships for farmers
application of pesticides and liquid fertilizers through Which of the statements given above is/are
drones will not only save the physical toil and hardship correct?
to the farmers but will also be quicker, saving time, (a) 1, 2, 3 and 5 only
(b) 2, 3 and 4 only
which can be utilized for more productive work. The
(c) 1, 3, 4 and 5
NAMO Drone Didi scheme will be a game-changer. It
(d) All of the Above
will usher in a new chapter of women’s empowerment
and effectively address multiple issues by providing an 35. An accurate clock shows the time as 9:00.
opportunity for rural women to be at the centre stage When the hour hand has moved by 150 o, then
of economic activity and rural prosperity. There is no what will be the time shown by the clock?
doubt that the scheme supported by the newly created (a) 1:00
network of Pradhan Mantri Krishi Samriddhi Kendras, (b) 2:00

which provide agricultural services under one roof, (c) 1:30


(d) 3:00
will prove to be a harbinger of modern-day agricultural
revolution and prosperity in a largely agrarian country
36. A group of 654 individuals is arranged in
like ours.
rows. Each row contains nine less individuals
than the row in front of it. What number of
33. Based on the above passage, the following rows are possible?
assumptions have been made: (a) 2
1. Rural areas still have gender-based (b) 4
inequalities. (c) 5

2. Faster deployment of technology is the (d) 6

only way to ensure agricultural growth.


37. If p, q. r and s are distinct single digit negative
Which of the assumptions given above is/are
integers, then what is the least possible value
correct?
of (p + q) (r + s) ?
(a) 1 only (a) 30
(b) 2 only (b) 25
(c) Both 1 and 2 (c) 24
(d) Neither 1 nor 2 (d) 21

www.visionias.in 11 ©Vision IAS

Telegram: UPSC_SARTHI
38. Shyam and Radhika paid a total of Rs. 63 for 41. If Q gets fewer coins than R, then which one
certain plates of poha and jalebi. If poha costs of the following is not necessarily true?
(a) P and Q together get at least four coins
Rs. 8 per plate and jalebi costs Rs. 5 per plate,
(b) Q and S together get at least four coins
then which one of the following statements is
(c) R and S together get at least five coins
correct? (d) P and R together get at least five coins
(a) We cannot determine whether they ate
more poha or jalebi. Directions for the following 4 (four) items:
(b) They ate same number of plates of poha Read the following four passages and answer the
items that follow each passage. Your answers to these
and jalebi.
items should be based on the passages only.
(c) They ate more jalebi than poha.
Passage – 1
(d) They ate more poha than jalebi. In common parlance, architecture is a study of forms:
plans, designs, motifs and how they have evolved.
Direction for following 3 (three) items: Architecture is also a medium to study society because

Read the information given below and answer the built spaces delineate communities and give them a
sense of belonging and cultural identity. Religion, in
questions that follow.
all time and space, has always been a major propeller
of architectural creations as of other artistic activities.
Ten coins are distributed among four people P, Q, R Overlap and interaction are the keys to understanding
and S such that one of them gets one coin, another gets Indian architecture. And since there is no linear
two coins, the third gets three coins and the fourth gets development in Indian architecture, the discipline
being a multiple discourse, we need to move away
four coins. It is known that Q gets more coins than P,
from the primacy of one region, period, dynasty, or
and S gets fewer coins than R.
patronage.

39. If the number of coins given to Q is twice the 42. Based on the above passage, the following
number of those given to P, then which one of assumptions have been made:
the following is necessarily true? 1. Indian architecture has served as a means
of aesthetic expression and a mirror that
(a) R gets an even number of coins
reflects the essence of society.
(b) R gets an odd number of coins
2. To comprehend the intricacies of Indian
(c) S gets an even number of coins architecture, one must delve into the
(d) S gets an odd number of coins intricate web of overlapping and
intermingling influences that have shaped
its unique form and character.
40. If R gets at least two more coins than S, then
Which of the assumptions given above is/are
which one of the following is necessarily true?
correct?
(a) Q gets at least two more coins than P
(a) 1 only
(b) Q gets more coins than S (b) 2 only
(c) P gets more coins than S (c) Both 1 and 2
(d) P and Q together get at least five coins (d) Neither 1 nor 2
www.visionias.in 12 ©Vision IAS

Telegram: UPSC_SARTHI
Passage – 2 44. Which one of the following statements best
During the recent period of heightened uncertainty, the reflects the most logical, rational, and crucial
emerging market economies (EMEs) were at the message implied by the passage?
receiving end of excess volatility in US dollar and (a) The evolution in the study of history is
bond yields. Given the systemic importance of the US based on the evolution of the study of
economy in the global financial system, spillovers culture from a social perspective.
from these fluctuations are not unexpected, especially
(b) Understanding culture from a historical
in the context of shifting expectations about the
perspective helps us understand the
monetary policy trajectory in the US and its
evolution of society.
fundamentals. In such a situation, the EMEs, which
(c) Expansion of the scope of history is
have their domestic dynamics and challenges, cannot
afford to be held hostage by international financial reflected in the evolution of society.
cycles. (d) Cultural studies now dominate history as
an academic stream.
43. Which of the following is the most logical
inference that can be made from the above Passage – 4
passage? Women have distinct nutritional requirements
(a) In an era of interconnected world, throughout their lives. Before pregnancy, women need
emerging market economies need to act to nutritious and safe diets to establish sufficient reserves
safeguard their unique interests. for pregnancy. During pregnancy and breastfeeding,
(b) The US economy has limited systemic energy, and nutrient needs increase. Meeting them is
importance in the global financial system. critical for women’s health and their child – in the
(c) Emerging market economies have no womb and throughout early childhood. But, in many
control over their domestic dynamics.
parts of the world, far too many women – especially
(d) Both (a) and (b)
adolescents and those who are nutritionally at-risk –
are not receiving the nutrition services they need to be
Passage – 3
healthy and give their babies the best chance to
History is no longer treated as merely a study of dates
survive, grow and develop.
and events and that too largely only political events. Its
scope has been widened to include many aspects of
life. These include the study of patterns of life, which 45. Which one of the following best reflects the
we call culture. Culture was once defined as that which crux of the passage?
relates to art, architecture, literature and philosophy. (a) For any particular woman, the nutritional
Now it includes all the activities of a society. requirement is the same throughout her
Therefore, the emphasis of history has shifted from the life.
study of only the upper groups of society to all levels (b) Pregnant women need specific diets, but
of society. It now covers information on kings and adolescents and nutritionally at-risk
statesmen as well as on ordinary people who make women do not.
history. It includes the study of art and architecture, of
(c) The nutritional status of women is poor in
the evolution of languages in India, literature and
many parts of the world, impacting their
religion. Now we do not look only at what was
health and their children's well-being.
happening at the aristocratic level of society. We also
(d) Governments need to ensure better
try to reconstruct the interests and concerns of people
facilities and nutrition for women and
at lower levels. This makes history more interesting
and helps us understand our society for the better. children.
www.visionias.in 13 ©Vision IAS

Telegram: UPSC_SARTHI
46. How many pairs of natural numbers are there 49. A sum of money at simple interest amounts to
such that the difference of their squares is 39? Rs. 215 in 2 years and to Rs. 255 in 3 years.
(a) One What must have been the principal sum?
(b) Two
(a) Rs. 135
(c) Four
(b) Rs. 125
(d) Three
(c) Rs. 100
47. Three views of a cube are given below. (d) Rs. 115

50. Consider the following sequence that follows


some logic:

Out of the four options given below, one of the a _ c d b _ d a c _a b d _ b c

options does not conform to the original cube.


Choose that option as your answer. The letters that should come in the gaps are
(a) (a) bcbc
(b) bbcc
(c) bcba
(d) bcda

(b)

Directions for the following 4 (four) items:


Read the following three passages and answer the
items that follow each passage. Your answers to these
(c)
items should be based on the passages only.
Passage – 1
Researchers analysing images from NASA’s James
Webb Space Telescope have found that galaxies in the
(d) early universe are often flat and elongated, like
surfboards and pool noodles – and are rarely round,
like volleyballs or frisbees. While most distant
galaxies look like surfboards and pool noodles, others
are shaped like frisbees and volleyballs. The

48. The number of squares that can be formed “volleyballs,” or sphere-shaped galaxies, appear the
from a set of five parallel lines intersecting most compact type on the cosmic “ocean” and were
another set of five parallel lines, is also the least frequently identified. The frisbees were
(a) 150 found to be as large as the surfboard- and pool noodle-
(b) 24
shaped galaxies along the “horizon,” but become more
(c) 30
common closer to “shore” in the nearby universe.
(d) 36
www.visionias.in 14 ©Vision IAS

Telegram: UPSC_SARTHI
51. Which of the following statements best 52. Which one of the following statements best
reflects the logical inference from the passage?
reflects the crucial message conveyed by the
(a) Galaxies in the early universe were
predominantly round, resembling author of the passage?
volleyball or frisbee shapes. (a) Self-reliance is essential for protecting
(b) Most distant galaxies in the early universe
specific firms and sectors against external
exhibit flat and elongated shapes, akin to
surfboards and pool noodles. competition.
(c) "Volleyball" galaxies are the most
(b) Self-reliance, as a response to global
frequently identified type in the cosmic
"ocean" of the early universe. economic challenges, may lead to greater
(d) Frisbee-shaped galaxies are primarily systemic instability.
found along the horizon of the universe.
(c) The intensifying US-China rivalry is a

Passage – 2 positive force for fostering economic


Self-reliance is different from protectionism. The
openness.
nominal goal is not to protect specific firms or sectors,
or to undercut others, but to build domestic resilience (d) The theory of international relations

in a less secure world. As an inward-looking strategy emphasizes the need for multiple global
of preservation, rather than an outward-looking
hegemons to ensure market accessibility.
program of punishment, it appears benign, even
sensible. But this is an illusion. Even if self-reliance is
an understandable response to a world that is moving 53. Based on the above passage, the following
away from economic openness, it risks fuelling even
assumptions have been made:
greater systemic instability. Today’s autarkic
tendencies are a symptom of the fading Pax 1. Erstwhile USSR lost is hegemon status
Americana. The intensifying US-China rivalry and the
after it resorted to protectionism as a
widening divide between democratic and authoritarian
regimes have increasingly impaired America’s ability reaction to the emerging challenges.
to keep the global market economy open. According to 2. The fading Pax Americana has enhanced
one theory of international relations, a trusted and
global economic openness.
committed hegemon that enforces global rules and
provides global public goods is a prerequisite to
keeping international markets open. When the
Which of the above assumptions is/are
predominant power no longer has the means or will to
play this role, markets suddenly become inaccessible. invalid?
The hegemon will use protectionism to contain rising (a) 1 only
challengers and preserve its global status while
(b) 2 only
reducing its international commitments. In response,
new challengers, like China today, will undermine the (c) Both 1 and 2
international system by challenging its legitimacy. (d) Neither 1 nor 2
www.visionias.in 15 ©Vision IAS

Telegram: UPSC_SARTHI
Passage – 3 56. Mr. X went to a restaurant for dinner where
People like Mahatma Gandhi, and Mother Teresa, who total bill was Rs. 500. The restaurant offers
have immensely contributed to society, were 30% discount on cash payment while 20%
psychologically powerful. One of the notable discount on card payment. Mr. X also gets
characteristics that they had is a well-developed 10% cashback on card payment after the bill is
conscience. Their ideas, words, and actions went paid. Which of the following statements is
together. Mahatma Gandhi thought that truth would correct?
always triumph; so, he spoke only the truth. Also, he (a) Cash payment is more profitable than card

carried out what he said. Likewise, Mother Teresa was payment.

concerned for the poor and the sick. She spoke for (b) Card payment is more profitable than cash

their welfare and dedicated her entire life to that cause. payment.

Likewise, many famous people all over the world have (c) Profit by cash payment is equal to profit
by card payment.
contributed to the welfare of society. All of them are
(d) He paid Rs. 360 to get maximum profit.
known for their integrity. Well-integrated people
contribute not only to their personal growth but also to
the growth and development of society.
57. Which of the following cannot be the ratio of
HCF to LCM of any two given natural
54. Based on the above passage, the following
numbers?
assumptions have been made:
(a) 1:15
1. Integrity predisposes a person to make
(b) 1:21
meaningful and constructive contributions
(c) 1:35
to society.
(d) 3:35
2. People who have a super-pious conscience
are always considered great souls.
58. Consider the following statements and the
Which of the assumptions given above is/are
question.
correct? I. W may or may not be older than V
(a) 1 only II. W is not older than V
(b) 2 only
(c) Both 1 and 2 X is older than Y, Z is younger than W and V
(d) Neither 1 nor 2 is as old as Y. Is Z younger than X?
(a) The question can be answered with the
55. A number N has 6 factors and lies between help of statement I alone.
250 and 350. What is the total number of (b) The question can be answered with the
possible values for N? help of statement II alone.
(a) 11 (c) Both statement I and statement II are
(b) 10 needed to answer the question.
(c) 12 (d) The statement cannot be answered even
(d) 9 with the help of both the statements.

www.visionias.in 16 ©Vision IAS

Telegram: UPSC_SARTHI
59. n is a natural number such that 0 < n < 101. 61. Which of the following statements can be
For how many values of n would the unit’s inferred from the above passage?
place of 2n + 8n end with a zero?
1. To achieve the ultimate objective of living
(a) 88
(b) 45 together, people should follow a social
(c) 50 order.
(d) 46 2. By overseeing group affairs and
promoting shared interests, the
60. In a school of 600 students, girls are 40%
more than boys. The average weight of girls is government facilitates the fulfilment of
50 kg and average weight of all students and citizens' joint goals.
boys is same. The average weight of all Select the correct answer using the codes
students is:
given below.
(a) 40 kg
(b) 80 kg (a) 1 only
(c) 60 kg (b) 2 only
(d) 50 kg (c) Both 1 and 2
(d) Neither 1 nor 2
Directions for the following 2 (two) items:
Read the following two passages and answer the items
that follow each passage. Your answers to these items Passage – 2
should be based on the passages only. Land is our basic resource. Throughout history, we
Passage – 1 have drawn most of our sustenance and much of our
The purpose for which people live together cannot be
fuel, clothing and shelter from the land. It is useful to
realized unless they are properly organized and accept
certain rules of conduct. The agency created to enforce us as a source of food, and as a place to live, work and
rules of conduct and ensure obedience is called the play. It has different roles. It is a productive economic
government. The government is also the focus of the factor in agriculture, forestry, grazing, fishing and
common purpose of the people occupying the definite
mining. It is considered a foundation for social prestige
territory. It is through this medium that common
policies are determined, common affairs regulated and and is the basis of wealth and political power. It has
common interests promoted. Without a government, many physical forms like mountains, hills, plains,
the people will lack cohesion and means of collective lowlands and valleys. It is characterized by the climate
action. There would be groups, parties warring
from hot to cold and from humid to dry. Similarly, the
associations and conditions of wars and chaos. So,
there is a need for common authority and order where land supports many kinds of vegetation. In a wider
people live. This is the prerequisite of human life. The sense, land includes soil and topography along with the
state cannot and does not exist without a government, physical features of a given location. It is in this
no matter what form a government may assume. The
context that land is identified closely with the natural
government is a must, though it may take any form. It
may have a monarchy like Bhutan or a republic as in environment. However, it is also regarded as space,
India. It may have a parliamentary form of government situation, a factor of production in economic processes,
like India and Great Britain or a presidential form of consumption goods, property and capital.
government like the United States of America.
www.visionias.in 17 ©Vision IAS

Telegram: UPSC_SARTHI
62. Which one of the following statements best 65. Consider the following question and the
reflects the crux of the passage? statements.

(a) The use of land as a resource has shaped Three friends P, Q and R are wearing hats,

the course of human political history. either black or white. Each person can see the
hats of the other two persons. What is the
(b) The land shapes our life in numerous
colour of P's hat?
manners, providing everything ranging
Statements:
from prestige to nutrition.
I. P says that he can see one black hat and
(c) Land use pattern remains the same across
one white hat.
the different physical forms.
II. Q says that he can see one white hat and
(d) Achieving prosperity in life is not possible one black hat.
without the economic ownership of land.

Select the correct answer using the codes


63. Rakesh was travelling from Mumbai to Goa. given below.
After travelling 23 km from Mumbai, a (a) The question can be answered with the
problem occurred in his car and its speed help of first statement alone.

became 4/5th of its original speed. Due to this, (b) The question can be answered with the
help of any one statement independently.
he reached Goa 50 minutes late. If that
(c) The question can be answered with the
problem had occurred after travelling 35 km
help of both statements together.
from Mumbai, then Rakesh would have
(d) The question cannot be answered even
reached Goa only 41 minutes late. Find the
with the help of both statements together.
original speed of car in km per hour.
(a) 25
66. Which of the following inequalities is/are
(b) 30 equivalent to the inequality -4 < 2x ≤ 2 ?
(c) 20 (a) x > -2 and x ≤ 1
(d) None of these (b) x < -2 or x ≥ 1
(c) x ≥ -2 and x < 1

64. A boy is asked to choose two numbers N 1 and (d) x ≤ -2 or x > 1

N2 at random without replacement from the


sequence 1, 2, 3, …….4n, where n is a natural 67. Ravi scored 20% marks in an examination and
failed by 120 marks. Rahul scored 40% marks
number. If the probability that (N 12 + N22) is
and scored 60 marks more than passing marks.
even is 11/23, then find the value of n.
Find the passing marks in the examination.
(a) 1
(a) 900
(b) 5
(b) 300
(c) 6
(c) 200
(d) 3 (d) 180
www.visionias.in 18 ©Vision IAS

Telegram: UPSC_SARTHI
68. In an over of 6 balls a batsman hits 3 Directions for the following 3 (three) items:
boundaries (four and sixes), takes two singles
Read the following three passages and answer the
and played a dot ball. Which of the following
cannot be the score of that batsman after the items that follow each passage. Your answers to these

over? items should be based on the passages only.


(a) 20
Passage – 1
(b) 18
(c) 16 A big issue regarding lack of energy is that students

(d) 12 have a small amount of time to study and complete

their homework. Many children walk long distances to


69. The figure given below consists of three
intersecting circles which represent the get to school from home, often leaving or returning in

applicants for the post of PA who are the dark. If, as is often the case, their house doesn’t
computer literate; who have knowledge of
have a source of energy, these students aren’t able to
office rules and regulations; and who have a
bachelor’s degree. study at home in the evening. They are often forced to

seek other sources outside their houses, sometimes at

gas stations or under streetlamps, to have enough light

to complete their schoolwork. Families that do have

energy at home often rely on kerosene lamps to

provide light, an expensive practice, that poses serious

health risks and oftentimes cannot be found locally.


Find the letter that represents all the applicants
who are computer literate and have knowledge
of office rules and regulations, but do not have 71. Which of the following statements best
a bachelor’s degree.
reflects the logical inference from the passage
(a) b
(b) c given above?

(c) d (a) Electricity distribution should be


(d) f
privatised.

70. In a half marathon conducted for girls, after (b) Without adequate and accessible
running for 30 minutes a runner was informed electricity, the overall development of
that she had completed only 60% of the
children is marred.
marathon. How much more time was the
runner supposed to run in this marathon? (c) There is a need to build renewable sources
(a) 50 minutes of energy for solving the issues of
(b) 25 minutes
affordability and accessibility.
(c) 20 minutes
(d) 40 minutes (d) Both (a) and (b)

www.visionias.in 19 ©Vision IAS

Telegram: UPSC_SARTHI
Passage – 2 Passage – 3

In the space of just a few years, gaming went from a


As human populations and demand for space continue
nerdy pastime to a multibillion-dollar industry. This
to grow, people and wildlife are increasingly
rise in popularity has resulted in a massive shift in the
interacting and competing for resources, which can
way video games are developed, produced, and
lead to increased human-wildlife conflict. Along with
distributed. Companies can’t expect to replicate their
other threats, human-wildlife conflict has driven the
success in Western markets with a “rinse and repeat”
decline of once-abundant species and is pushing others
of existing strategies for new geographies. Different
to the brink of extinction. But the human-wildlife
aspects of the game must be tailored for local playing
conflict issue has far-reaching impacts beyond the
conditions as though the game was built from the
wildlife and communities immediately affected by it.
ground up for that market. The internet allows people
With human-wildlife conflict centred around the
to build communities through gaming forums and
interaction between wildlife and humans, human-
groups, exposing far larger populations to a wider
wildlife coexistence is strongly linked and important to
variety of video games. Companies that want to access
sustainable development activities. If not effectively
(and profit from) the fast-growing gaming consumer
managed, human-wildlife conflict has the potential to
base need to be prepared to provide instruction,
negatively affect these activities and conservation
support, and moderation teams in multiple local
much more broadly.
languages.

72. Which one of the following statements best


73. Which one of the following statements best
reflects the crux of the passage?
reflects the crux of the passage?
(a) Increase in human-wildlife conflict is an
(a) Gaming companies need AI for
indication of transition towards sustainable
development and distribution of games.
development.
(b) The perspective of Western markets and
(b) Success of sustainable development
other regions about gaming is completely
activities depends on the reduction of
different.
human-wildlife conflicts.
(c) Globalization of games through the
(c) To reduce human-wildlife conflict, it is
internet for profit-making needs
essential to decrease the rate of population
localization.
growth.
(d) Gaming companies are more successful in
(d) Human beings and animals are both
Western markets as compared to new
equally affected by human-wildlife
geographies.
conflicts.

www.visionias.in 20 ©Vision IAS

Telegram: UPSC_SARTHI
74. There are six employees in a company. Some 77. Choose the group which is different from the
hints about their salary are given below. others.
Consider the following statements and answer (a) 13, 05, 35, 11
the question that follows. (b) 02, 09, 17
1. Salary of Ajay is more than that of Bijay. (c) 34, 29, 41, 07, 14
2. Chetan and David earn the same amount. (d) 76, 24, 16, 64, 04, 32
3. Eshaan earns the least.
4. Firoz earns less than David.
78. If 12 3 45 = 6, 67 31 56 = 1, 54
5. Firoz earns more than Ajay.
How many statements given above are 23 89 = 4, then what is the value of 73

required to determine the highest earning 34 21 ?


person(s)? (a) 13
(a) Only 1, 2 and 4 (b) 2
(b) Only 2, 3, 4 and 5 (c) 7
(c) Only 2, 4 and 5 (d) 32
(d) All five

79. Three Statements followed by two


75. A shopkeeper makes 30% profit by selling an Conclusions are given below. You have to
item X mixed with item Y at a price of Rs. 250 take the Statements to be true even if they
per kg. Item Y comes free of cost, and item X seem to be at variance from the commonly
mixed with item Y is sold at the price of pure
known facts. Read all the Conclusions and
item X. Find the ratio of Y to X in the mixture.
then decide which of the given Conclusions
(a) 4 : 5
logically follows/follow from the Statements,
(b) 5 : 9
disregarding the commonly known facts.
(c) 3 : 10
Statement-1: All pen are pencils.
(d) 7 : 8
Statement-2: All pencils are erasers.
Statement-3: All erasers are sharpeners.
76. In a village, the ratio of the number of males to
females is 4:5. The ratio of literate males to
Conclusion-I: All pen are sharpeners.
literate females is 6:7. The ratio of the number
Conclusion-II: All sharpeners are pencils.
of illiterate males to illiterate females is 3:4. if
2400 males in the village are literate, then find
Which of the above Conclusions logically
the difference between the number of males
and females in the village. follows/follow from the given Statements?

(a) 1280 (a) Only Conclusion-I

(b) 1320 (b) Only Conclusion-II

(c) 1200 (c) Neither Conclusion-I nor Conclusion-II

(d) 1000 (d) Both Conclusion-I and Conclusion-II


www.visionias.in 21 ©Vision IAS

Telegram: UPSC_SARTHI
80. If the angles of a triangle are in the ratio 2:3:4,
then which of the following statements is true?
(a) No two sides of the triangle are of equal
length.
(b) One of the angles of the triangle is obtuse.
(c) The triangle is a right angled triangle.
(d) The triangle may be equilateral.

Copyright © by Vision IAS


All rights are reserved. No part of this document may be reproduced, stored in a retrieval system or transmitted
in any form or by any means, electronic, mechanical, photocopying, recording or otherwise, without prior
permission of Vision IAS.
www.visionias.in 22 ©Vision IAS

Telegram: UPSC_SARTHI
VISIONIAS
www.visionias.in
ANSWERS & EXPLANATION
CSAT 2024 – ABHYAAS TEST – 1 – 5058

1 (a)
It is given that:
1 X 6 2
5 8 Y 9
Z 4 0 1
Since, X, Y and Z are single-digit odd prime integers so the possible values which X, Y and Z can have are 3, 5
and 7.
1 5 6 2
5 8 3 9
7 4 0 1 (Carry 1)
Step 1: After adding 9 + 2 we get 11 (i.e. 1 at unit place with carry 1).
Step 2: Now to get 0 at ten's place with carry 1, Y must be 3, i.e. (1 + 6 + 3 = 10).
Step 3: Now, to get 4 at hundredth place with carry 1, X must be 5, i.e. (1 + 5 + 8 = 14).
Step 4: Z will be 1 + 1 + 5 = 7.
Hence, X + Z = 5 + 7 = 12
So, option (a) is the correct answer.

2 (b)
Assumption 1 is not correct: The passage mentions that the primary goal of the Mughals was “…to reduce the
burden of work and expenses for the Mughal administration”. Fixing a village sum might simplify tax collection
for administration, but wouldn't necessarily empower individual peasants or promote local autonomy. Hence,
assumption 1 is not correct.
Assumption 2 is correct: The passage states that the Mughal administration implemented several systems of land
assessment which were designed to “…reduce the burden of work and expenses for the Mughal administration”.
This suggests that the Mughals were concerned with the efficiency of their revenue collection system. The passage
further says that as per one of the leading historians, one major aim of the Mughal administration was to take
away the bulk of the peasant’s surplus. This suggests that the Mughals were primarily interested in maximizing
their revenue collection. Hence, assumption 2 is correct.

3 (b)
Option (a) is not correct: While the passage emphasizes the environmental impact of various development
activities, it does not suggest outright ban on development works. The focus is on finding ways of responsible
development works and minimization of resource consumption. Hence, option (a) is not correct.
Option (b) is correct: This message captures the core of sustainable development and its crucial role in the
context of limited resources and ongoing development activities. The passage unequivocally acknowledges that
1 www.visionias.in ©Vision IAS

Telegram: UPSC_SARTHI
the utilization of environmental resources has been adversely impacted by development activities spanning diverse
sectors. It is reflected in the lines, "The declaration of the United Nations Conference on Human Environment
recognized the finite nature of resources available on earth and emphasized its judicious use to remain available
for future generations". Hence, option (b) is correct.
Option (c) is not correct: While the efforts of United Nations and WCED played a significant role in raising
awareness and proposing concrete actions, the passage mentions that the idea of sustainable development existed
ever since human beings started using the natural resources for their developmental needs. The UN and WCED
played a contributing role; however, they did not invent this concept. Hence, option (c) is not correct.

4 (b)
Assumption 1 is correct: The passage describes how one environmental change triggers another, leading to
increasingly severe consequences. The passage says, “On our current path, ice cap melt will cause sea levels to
rise to levels where many major cities will be at very high risk of flooding, and natural disasters will cause
damage to our communities at catastrophic levels on a much more regular basis”. This assumption emphasizes
the systemic nature of environmental problems and the need for holistic solutions that address their
interconnectedness.
Assumption 2 is not correct: While the passage highlights the severity of environmental changes, it doesn't
necessarily imply that all damage is permanent and beyond repair. The passage does not mention the
irreversibility of environmental changes. Hence, statement 2 is not correct.
Assumption 3 is not correct: The passage mentions how growing population exacerbates environmental
problems, but this does not mean that the growing population is the root cause of the environmental issues as
mentioned in the option. Furthermore, the author does not make any distinction between urban and rural
population. So, this option is not the correct assumption as per the passage.

5 (c)
Let’s choose number 10 to work with.
If we subtract 3 from this number and then multiply the result by 4, we get 4(10 - 3) = 28
Let the number to be subtracted be x.
Then we get:
4×10 – x = 28
Or 28 = 40 – x
Or -12 = -x
Or x = 12
So, option (c) is the correct answer.

6 (a)
315 = 3 x 3 x 5 x 7
Its factors are 3, 5, 7, 9, 15, 21, 35, 105…..
So, p1 + p2 = 3, 5, 7, 9, 15, 21, 35, 105….. all of which are odd.
Since p1 + p2 is odd, and p1 < p2 ; so, p1 must be 2. Only one value of p1 is possible.
So, option (a) is the correct answer.

2 www.visionias.in ©Vision IAS

Telegram: UPSC_SARTHI
7 (b)

MDVJH is obtained from GIVEN first by writing it backwards and then writing preceding letter in alphabetical
series for the first two, successive letter for the last two and leaving middle letter unchanged.
So, SHARK would be coded as JQAIT.
So, option (b) is the correct answer.

8 (d)
Let n1 be the number of pairs of black socks, and n2 be the number of pairs of blue socks.
Let x be the cost price per pair of blue socks. Then, 2x must be the cost price per pair of black socks.
As per the question,
1.5 (2n1x + n2x) = 2n2x + n1x
Or n1/n2 = 1/4
Since n1 = 4, so n2 = 16
So, option (d) is the correct answer.

9 (d)
Sum of 6 numbers, a + b + c + d + e + f = 6 × 36 = 216
Ratio would be the maximum when d is maximum and c is minimum.
To maximize d, all other numbers should be the least possible.
Least possible values of a, b, c are 1, 2, 3.
Least possible values of e and f are d+1, d+2.
Therefore, 1 + 2 + 3 + d + (d + 1) + (d + 2) = 216
Or d = 69
Required ratio = 69/3 = 23:1
So, option (d) is the correct answer.

10 (c)
216 = 63
So, n = 6
The number of smaller cubes with no coloured face, N = (n – 2)3 = (6 – 2) 3 = 43 = 64.
So, option (c) is the correct answer.

3 www.visionias.in ©Vision IAS

Telegram: UPSC_SARTHI
11 (a)
Inference 1 is correct: The passage highlights media's role in transmitting diverse perspectives to policymakers.
They bring to light the opinions of various groups, allowing decision-makers to consider viewpoints from diverse
perspectives. Beyond simply informing, the media also wields the power to pressure authorities into action. By
promoting public debates and raising awareness about people's needs and demands, they can create a sense of
urgency and push policymakers to respond promptly. Hence, inference 1 is correct.
Inference 2 is not correct: While the passage highlights the historical role of media in furthering bourgeois
democracy, it doesn't necessarily imply that this role is intrinsic and present even today. The passage offers a
particular historical viewpoint on the nature of the Fourth Estate. Based only on this context, it might be incorrect
to draw a broader generalization about the inherent bias of the media. Hence, inference 2 is not correct.
Inference 3 is not correct: The passage primarily discusses the media's impact on public opinion and social
management. The author, nowhere, claims a complete replacement of the democratic ethos. In fact, the passage
emphasizes the complementary and mutually strengthening roles of democracy and media in overall development.
Hence, statement 3 is not correct.

12 (c)
Option (a) is not correct: While the passage highlights the emergence of the working class and its early
movements in the context of industrialization, it doesn't imply that all social movements fall under this category.
There could be other types of social movements also. Hence, option (a) is not correct.
Option (b) is not correct: The passage describes the evolution of the working class from holding unorganized
actions to organized protests through associations. However, the passage does not mention that success of social
movements depends only upon their level of organization. There could be many other contributing factors. Hence,
option (b) is not correct.
Option (c) is correct: The statement is correct in the context of the passage – “The actions of the working class
in the earliest stage were sporadic and unorganized and hence were mostly ineffective.” Hence, option (c) is
correct.
Option (d) is not correct: Though the passage has highlighted working class protests, we cannot deduce from
this that study of social movements in India is labour centric. Hence, option (d) is not correct.

13 (a)
Option (a) is correct: The passage highlights childhood as the most critical phase for internalizing values, beliefs,
and behaviour patterns from family - the primary socializing agents. The passage says, “childhood is the most
important stage in the process of socialization”. Socialization is continuous, occurring throughout life, not just in
childhood. Just that childhood is the most important of all phases of life. Hence, option (a) is correct.
Option (b) is not correct: The passage says, “We may also mention that all learning is not socialization since
some of what one learns may not be relevant or necessary for participation in given social roles”. It clearly
suggests that not everything learned plays a direct role in socialization. Therefore, it would be incorrect to say that
all social knowledge will aid our socialization. Hence, option (b) is not correct.
Option (c) is not correct: The passage mentions, “In some societies, the education and socialization of the young
takes place without extensive formal educational institutions”. It can be implied that though some societies may
teach socialization to their young ones without any formal educational institutions, it does not entirely reject the
requirement of formal education system. Formal education system could help in other fields. Hence, option (c) is
not correct.
Option (d) is not correct: The passage says, “The parents can be viewed as the socializing agents.” It neither
compares the role of parents with any other socializing agent, nor does it state that parents are the only socializing
agent for children. Therefore, it is not possible to conclude that parents are the most important socializing agents
for children. Hence, option (d) is not correct.

4 www.visionias.in ©Vision IAS

Telegram: UPSC_SARTHI
14 (d)
Assumption 1 is not correct: This assumption oversimplifies the complex dynamics of social change. The
assumption implies a passive acceptance of British influence through education. The British were interested in
bringing change in Indian society for their own benefit primarily. The passage says, “To serve their interests the
British rule introduced a new western education, new means of transport and communication, new technology and
a new system of judiciary”. Also, the British initiated reforms to bring about social change, but the pivotal nature
of those reforms is not covered in the passage. Therefore, attributing a positive social change to colonial education
cannot be held as a correct assumption. Hence, statement 1 is not correct.
Assumption 2 is not correct: The author says, “In the initial stages the British rule influenced the port and coastal
cities”. The passage does not explicitly mention or indicate that training the dwellers of coastal cities served the
British interests through exports from India. Hence, statement 2 is not correct.

15 (d)
A, B, C are brothers of each other. D is their sister, but we cannot say anything about E’s gender (whether E is
male or female and hence brother or sister is unknown).
Hence, option (d) is correct.

16 (c)
Here, on dividing 55 by natural numbers (starting from 1), the sequence of remainders will be:
55 ÷ 1 = Remainder 0
55 ÷ 2 = Remainder 1
55 ÷ 3 = Remainder 1
55 ÷ 4 = Remainder 3
55 ÷ 5 = Remainder 0
55 ÷ 6 = Remainder 1
55 ÷ 7 = Remainder 6
55 ÷ 8 = Remainder 7
Hence, X = 1.
Hence, option (c) is correct.

17 (c)
Their final positions would be:

Hence, option (c) is correct.

5 www.visionias.in ©Vision IAS

Telegram: UPSC_SARTHI
18 (b)
Let the number of hired men, transgenders and women be 8x, 7x and 4x respectively.
So, 8x + 7x + 4x = 76
⇒ 19x = 76
∴ x = 4.
So, the number of hired men, transgenders and women are 32, 28 and 16 respectively.
Let a man does 1 work in 2 days.
So, a transgender does 1 work in 3 days.
So, in 1 day, a man does 1/2 work;
And in 1 day, a transgender does 1/3 work.
So, in 1 day, a woman does (3/4) (1/3) = 1/4 work.
So, the working efficiency (work/day) of man : transgender : woman = 1/2 : 1/3 : 1/4
= 1/2 x 12, 1/3 x 12, 1/4 x 12 [LCM(2,3,4) = 12]
= 6 : 4 : 3.
So, the ratio of work done in a day by 32 men, 28 transgenders and 16 women= (32 x 6): (28 x 4): (16 x 3) =
12 : 7 : 3.
Hence, the daily wage of Rs 8800 should be divided in this ratio.
Money eamed by 28 transgenders in a day = 7/22 x 8800 = Rs 2800

So, daily wage of a transgender should be Rs 100.


Hence, option (b) is correct.

19 (b)
Possible Venn diagram:

So, both the conclusions follow.


Hence, option (b) is correct.

20 (c)
Let, total guests be 100.
Number of women = 60
Number of men = 40
Total number of married couple guests = 80, i.e. (40 men and 40 women).
Thus, we can say that all men are married. Hence, statement 1 is correct.
Now, out of a total of 60 women, 40 are married. Hence 60 – 40 = 20 women are unmarried.
20 = x% of 60

6 www.visionias.in ©Vision IAS

Telegram: UPSC_SARTHI
or x = (20 × 100)/60
or x = 100/3 = 33.33%
Hence, statement 2 is also correct.
Hence, option (c) is correct.

21 (b)
Since, x, y and z are natural numbers divisible by 3. So x, y and z can be 3, 6, 9, 12, 15, and 18.
Triplets of natural numbers that satisfy the given condition are:
3, 6, 12 - this can be arranged in 6 ways i.e. (3612, 3126, 6123, 6312, 1263, 1236).
3, 3, 15 - this can be arranged in 3 ways i.e. (3315, 3153, 1533).
6, 6, 9 - this can be arranged in 3 ways i.e. (669, 696, 966).
9, 9, 3 - this can be arranged in 3 ways i.e. (993, 939, 399).
Fifteen triplets satisfy the given condition.
Hence, option (b) is the correct answer.

22 (b)
For the year to have the same calendar as 2020, you need the sum of the number of odd days to be divisible by 7.

∴ 1st January 2025 will fall on Monday.


Hence, option (b) is the correct answer.

Alternate Method:
5-year pattern: If there are two leap years between dates, the date shall fall on the same day after 5 years.
In the question the given date is 1st January 2020 and after five years it will be 1st January 2025. During this period
we see that 2020 and 2024 are the two leap years.
Hence, we can say that on 1st January 2025, 'X' will celebrate his birthday again on the same day, i.e. on Monday.

23 (a)
Assumption 1 is correct: The passage in lines “While achieving food sufficiency in production, India still
accounts for a quarter of the world’s hungry people and is home to over 190 million undernourished people”,
highlights that despite being the largest source of livelihood and achieving grain self-sufficiency, India still houses
a significant portion of the world's hungry and undernourished population. This suggests a potential disconnect
between overall production and equitable distribution. Hence, assumption 1 is correct.
Assumption 2 is incorrect: The assumption may be correct otherwise, but is beyond the context of the passage.
The passage says, “The increased feminisation of agriculture is mainly due to increasing rural-urban migration by
7 www.visionias.in ©Vision IAS

Telegram: UPSC_SARTHI
men, the rise of women-headed households and growth in the production of cash crops which are labour intensive
in nature”. Though it is an unintended outcome, it may lead to women empowerment. So, it would be incorrect to
say with surety that it will not lead to women empowerment. Hence, assumption 2 is not correct.

24 (a)
Inference 1 is correct: The passage starts off by highlighting contemporary advancements like agricultural
modernization and globalization, and then points out the persistence of traditional concepts of gender roles. The
specific example of women leaving their families after marriage indicates a continuation of the traditional
patriarchal structures. The statement about unpaid and undervalued domestic work suggests a lack of recognition
for women's contributions within the household, reinforcing traditional gender divisions. Hence, inference 1 is
correct.
Inference 2 is not correct: The passage mostly addresses internal dynamics in Indian society, emphasizing how
conventional gender roles continue to exist despite social progress. It doesn't directly draw parallels with other
societies. Hence, inference 2 is not correct.

25 (c)
The sum of the first 50 positive even integers: 2 + 4 + 6 + 8 + . . . + 100.
These numbers can be regrouped into 25 pairs of numbers, each of which has a sum of 2 + 100 = 102.
Therefore, their sum is 25 × 102 = 2,550.
Hence, option (c) is the correct answer.
26 (b)
After considering all the given statements, we can deduce the following order:
Niraj > Om > Parvati (37) > Laxman > Manish (34) > Kavita
Hence, option (b) is the correct answer.

27 (b)
The revenue is equal to the number of items sold times the price per item.
If the restaurant typically sells n sandwiches per day, but today sold 50% more, it sold 1.5n sandwiches. If the
price p was reduced by 30%, then today’s price is 0.70p.
Therefore, total revenue = 1.5n × 0.70p = 1.05np.
Hence, option (b) is the correct answer.

28 (c)
We need to find the relation of Erapalli with Bhavna.
We must use statements 1 and 3, as Bhavna and Erapalli have not been mentioned in any other statement.
Option (a): Statements 1, 2 and 3 alone are not sufficient, as we cannot find a link between Bhavna and Erapalli in
the statements.
Option (b): Statements 1. 3 and 4 alone are not sufficient, as we cannot find a link between Bhavna and Erapalli in
the statements.
Option (c): Using all the four statements, we get the following family tree:

8 www.visionias.in ©Vision IAS

Telegram: UPSC_SARTHI
So, Erapalli is the brother-in-law of Bhavna.
Hence, option (c) is the correct answer.

29 (c)
Let’s assume that the cost of living in 1980 was Rs.100.
If this increased by 20% from 1980 to 1990, then the cost of living in 1990 = 1.20 × 100 = Rs. 120.
If the increase from 1980 to 2000 was 50%, then the cost of living in 2000 = 1.50 × 100 = Rs.150.
The percent increase from 1990 to 2000 is therefore [(150 – 120) / 120] × 100 = (30/120) × 100 = 25%
Hence, option (c) is the correct answer.

30 (c)
B + C + D = 146 ......................... (i)
C + D + A = 198 ........................... (ii)
A + B + C = 128 ........................... (iii)
D + A + B = 164 ............................ (iv)
By adding all equations we get,
3 × (A + B + C + D) = 146 + 198 + 128 + 164 = 636
or A + B + C + D = 636/3 = 212 ................ (v)
From (i) and (v) we get,
A + 146 = 212
or A = 66
From (ii) and (v) we get,
B + 198 = 212
or B = 14
From (iii) and (v) we get,
D + 128 = 212
or D = 84
Now, from equation iii we get,
∵ B = 14 and A = 66 and A + B + C = 128
⇒ C = 128 – (14 + 66) = 48
Therefore, we get A = 66, B = 14, C = 48 and D = 84.

9 www.visionias.in ©Vision IAS

Telegram: UPSC_SARTHI
Therefore, we can say that B is the smallest number and B = 14. Hence, conclusion l is true.
D is the largest number and D = 84. Hence, conclusion II is false.
A is not the largest number but A = 66. Hence, conclusion lll is true.
Hence, only conclusions I and III follow.
Hence, option (c) is the correct answer.

31 (c)
Option (a) is incorrect. The following line from the passage indicates that the world has failed to gain much since
the implementation of the Paris Pact - “But that only indicates how little UNFCCC meets have achieved since the
Paris Pact of 2015.” This answer options mentions just the opposite. Hence, it is incorrect.
Option (b) is incorrect. The problems faced in framing an acceptable language on fossil fuel elimination reflect
the contradictions between countries as far as this issue is concerned. Hence, it would be incorrect to say that all
the countries have unanimously agreed on the elimination of fossil fuels.
Option (c) is correct. The entire passage illustrates the failures of climate talks to reach a consensus to undertake
actions that can mitigate climate change. The declaration addressing concerns of India and China, but
disappointing small island states is a testimony to this lack of consensus. This answer option aptly captures this
argument.
Option (d) is incorrect. The lines, “In the run-up to COP28, it was apparent that putting the matter on the back
burner had pushed the world perilously close to the threshold beyond which climate change becomes irreversible”
suggest that we are close to the point where climate change would become irreversible. However, it is not
irreversible yet. Hence the option is incorrect.

32 (b)
Option (a) is incorrect. The passage nowhere mentions Atal Setu being the peak of the development of physical
infrastructure. It merely highlights the many advantages of building Atal Setu. Hence, this option is incorrect.
Option (b) is correct. The central theme of the passage is to highlight that the construction of Atal Setu is
important in terms of developing the global image of India as an investment destination. It can be validated in the
lines “It is indeed another key step in India's bid to transform not only its physical infrastructure but also its global
image as an investment destination.” This global image can be leveraged as an opportunity by India for its
development. Hence, this statement is correct.
Option (c) is incorrect. The given statement is correct, but it does not reflect the central idea presented in the
passage. The author clearly mentions that the advantages go much beyond the benefits of it being a physical
infrastructure. This answer option captures only the physical infrastructure part. Therefore, this statement does not
quite qualify to be the crux of the passage.
Option (d) is incorrect. This is an irrelevant statement because the passage nowhere mentions the cutting of
forests or the environmental impact of building Atal Setu. Hence, the option is incorrect.

33 (a)
Assumption 1 is correct. The passage shows the significance of the NAMO Didi Drone Scheme in terms of new
work opportunities for women. The line “This will go a long way in making our agrarian family culture more
equitable and stronger” implies that gender-based inequalities are still prevalent in the rural landscape. Hence, the
assumption is valid.
Assumption 2 is incorrect. The passage highlights the importance of technology in agricultural progress.
However, the passage does not claim this to be the only way of ensuring agricultural growth. There could be
various other factors in the growth story of this sector. Hence, the second assumption is invalid.

10 www.visionias.in ©Vision IAS

Telegram: UPSC_SARTHI
34 (c)
Statements 1, 3, 4 and 5 are correct. The lines “The application of pesticides and liquid fertilizers through
drones will not only save the physical toil and hardship to the farmers but will also be quicker, saving time, which
can be utilised for more productive work” clearly validate statements 1, 4 and 5. The line “A woman drone pilot
can play a vital role in increasing agricultural productivity and efficiency” validates statement 3.
Statement 2 is incorrect. The passage nowhere mentions or implies that drone technology could make agriculture
‘completely’ self-sufficient. It would be an extreme presumption. Therefore, this statement is not correct.

35 (b)
Hour hand covers 360o in 12 hours.
∴ Time taken to cover angle of 150o = (12/360o) × 150o = 5 hours
9 o’clock + 5 hours = 2 o’ clock
Therefore, clock will show 2 o’ clock after hour hand covers an angle of 150o.
Hence, option (b) is the correct answer.

36 (b)
Let number of individuals in front row be x.
So, number of individuals in preceding rows will be (x – 9), (x – 18), (x – 27) and so on.
Suppose there are 4 rows, then
x + (x – 9) + (x – 18) + (x – 27) = 654
⇒ 4x – 54 = 654
⇒ 4x = 654 + 54 = 708
⇒ x = 708/4 = 177
On checking with other options we do not get perfect integer value for x.
Hence, option (b) is the correct answer.

37 (d)
Single digit negative integers are : -1, -2, -3, -4, -5, -6, -7, -8, and -9. To minimize the value of (p + q) (r + s), we
need to ensure that we multiply the smallest numbers possible (magnitude-wise). So, we will only consider the
numbers -1, -2, -3, and -4. Also, we should ensure that the numbers being multiplied are as far in value as
possible.
So, the required answer = {-1 + (-2)} {-3 + (-4)} = -3 × -7 = 21
Hence, option (d) is correct.

38 (a)
They paid a total Rs. 63.
Cost of poha per plate = Rs. 8
Cost of jalebi per plate = Rs. 5
Amount paid for poha will be in the multiple of 8, i.e. 8, 16, 24, 32, 40, 48, 56 and so on.
Amount paid for Jalebi will be in the multiple of 5, i.e. 5, 10, 15, 20, 25, 30, 35 and so on.
By hit and trial:

11 www.visionias.in ©Vision IAS

Telegram: UPSC_SARTHI
Case 1:
(Rs. 8 × 6) + (Rs. 5 × 3) = Rs. (48 + 15) = Rs. 63
Case 2:
(Rs. 8 × 1) + (Rs. 5 × 11) = Rs. (8 + 55) = Rs. 63
∴ They purchased 6 plates of poha and 3 plates of jalebi OR 1 plate of poha and 11 plates of jalebi.
Hence, option (a) is correct.

39 (d)
It is given that Q > P and R > S.
The distribution of coins can be done in two ways:
QPRS
2143
4231
In both the cases, S gets an odd number of coins.
Hence, option (d) is correct.

40 (b)
The possible arrangements are:
RSQP
3142
4132
4231
Looking at the possible distribution of coins, we can deduce that option (b) must always be true
Hence, option (b) is correct.

41 (a)
The possible arrangements are:
RSQP
4321
4231
4132
Looking at the possible distribution of coins, we can deduce that option (a) need not always be true.
Hence, option (a) is correct.

42 (c)
Assumption 1 is correct: The passage emphasizes that architecture is not just about plans and designs, but also
about communities, belonging, and cultural identity. Buildings help in delineating communities; this suggests their
role in shaping social structures and providing a sense of place. The connection with religion further highlights the
cultural and societal significance of architecture in India. Hence, assumption 1 is correct.

12 www.visionias.in ©Vision IAS

Telegram: UPSC_SARTHI
Assumption 2 is correct: The passage stresses the importance of acknowledging overlaps and interactions in
understanding Indian architecture. The lines from the passage “Overlap and interaction are the keys to
understanding Indian architecture” clearly validate this assumption. Hence, assumption 2 is correct.

43 (a)
Option (a) is correct. This option is correct because the passage emphasizes the impact of the US monetary
policy trajectory on EMEs. The lines from the passage, “In such a situation, the EMEs, which have their domestic
dynamics and challenges, cannot afford to be held hostage by international financial cycles” reflect that EMEs
need to safeguard their interests. Therefore, one can infer that EMEs, which have their own set of challenges, must
act to safeguard their interests.
Option (b) is incorrect. This answer option contradicts the author who emphasises the systemic importance of the
US economy in the global financial system. It is clearly evident from the lines: “Given the systemic importance of
the US economy in the global financial system.”
Option (c) is incorrect. The passage does not imply this. It only lays out the impact the EMEs face due to
changes in US policy. Also, the passage does not discuss the control of EMEs on their respective domestic
dynamics. Therefore, this option cannot be inferred from the passage.
Option (d) is incorrect as option (b) is incorrect.

44 (a)
Option (a) is correct: The passage argues that studying history makes us understand wider aspects of life, and
offers a more complete picture of humanity and its evolution. This understanding, in turn, empowers us to better
comprehend our society and its complexities. It is no longer only a political record of events, but a broader
outlook towards society. It can be understood from the following examples:
From elite to inclusive: The emphasis now includes the viewpoints and experiences of common people from all
social classes, rather than only the wealthy, monarchs, and statesmen.
From limited to diverse: To provide a more thorough knowledge, the scope of history today includes a variety of
social life-related aspects, such as literature, art, architecture, language change, and religion.
It can be implied that the scope of history has expanded from being only political to societal, now. Hence, option
(a) is correct.
Option (b) is not correct: The passage is about the expansion in the scope of history through culture as suggested
in option (a), and not vice versa as stated in this answer option. Therefore, to say that understanding culture from a
historical perspective helps in better understanding of society would be incorrect as it is not based on the
information provided in the passage. Hence, option (b) is not correct.
Option (c) is not correct: The central theme in the statement is the evolution of society, and how it is reflected in
the expansion of history. However, the passage does not mention anything related to evolution in society. It
focuses on expansion in the scope of history based on the expansion in the study of culture by including societal
perspectives. So, this option is not the crucial message of the passage. Hence, option (c) is not correct.
Option (d) is not correct: The passage says, “History is no longer treated as merely a study of dates and events
and that too largely only political events. Its scope has been widened to include many aspects of life. These
include the study of patterns of life, which we call culture”. The passage further mentions, “Culture was once
defined as that which relates to art, architecture, literature and philosophy. Now it includes all the activities of a
society”. The author only implies that cultural aspects of society are now studied in the discipline of history. It,
nowhere, implies that cultural studies are now dominating history as an academic stream. Hence, Option (d) is
not correct.

45 (c)
Option (a) is incorrect because it suggests that for any particular woman, the nutritional requirement is the same
throughout her life, which contradicts the essence of the lines “Before pregnancy, women need nutritious and safe
diets to establish sufficient reserves for pregnancy. During pregnancy and breastfeeding, energy, and nutrient

13 www.visionias.in ©Vision IAS

Telegram: UPSC_SARTHI
needs increase.” So, even for a particular woman, the dietary requirements change depending on whether she is in
pre-natal, ante-natal or post-natal stage.
Option (b) is incorrect because the passage highlights that women have distinct nutritional requirements
throughout their lives in the very first line itself. Also, the passage nowhere implies that adolescents and
nutritionally at-risk women do not need specific diets. So, this option is not correct.
Option (c) is correct because the passage emphasizes that many women, especially adolescents and those
nutritionally at-risk, are not receiving the nutrition services they need, impacting their health and their children's
well-being.
Option (d) is incorrect because the context of the role of government is not discussed in the passage. Hence, this
option is beyond the scope of the passage and is not the best crux.

46 (b)
Let, the numbers be x and y.
According to question,
x2 – y2 = 39
Now, for 1st pair:
⇒ (x – y) (x + y) = 39 × 1
On comparing, we get
x – y = 1 and x + y = 39
On solving, we get
x = 20 & y = 19
So, one pair = (20, 19)
Similarly for 2nd pair:
⇒ (x – y) (x + y) = 13 × 3
So, on comparing, we get
x – y = 3 and x + y = 13
On solving, we get
x=8&y=5
So, another pair = (8, 5)
Hence, there are only two such pairs.
Hence, option (b) is correct.

47 (c)
Considering first and second cubes:
A is common in both the views. By applying the clockwise rule, we get the following pair of opposite
faces:
B – D ; C – E and A - F

So, we can conclude that E & C must be opposite to each other.


In option (c), E & C are shown as adjacent faces. So, it does not confirm to the original cube.
Hence, option (c) is correct.

14 www.visionias.in ©Vision IAS

Telegram: UPSC_SARTHI
48 (c)
When five parallel lines intersect another set of five parallel lines, we get the following figure:

Total number of squares in the above figure is:


Size Rows Columns Total
1×1 4 4 16
2×2 3 3 9
3×3 2 2 4
4×4 1 1 1
Total number of squares = 30
Alternate method:
The figure may be labelled as shown.

The simplest squares are ABGF, BCHG, CDIH, DEJI, FGLK, GHML, HINM, IJON, KLQP, LMRQ, MNSR,
NOTS, PQVU, QRWV, RSXW and STYX, i.e. 16 in number.
The squares composed of four components each are ACMK, BDNL, CEOM, FHRP, GISQ, HJTR, KMWU,
LNXV and MOYW, i.e. 9 in number.
The squares composed of nine components each are ADSP, BETQ, FIXU and GJYV, i.e. 4 in number.
There is one square AEYU composed of sixteen components.
So, there are 16 + 9 + 4 + 1 = 30 squares in the given figure.
Hence, option (c) is correct.

49 (a)
S.I. for 1 year = Rs. (255 - 215) = Rs. 40.
S.I. for 2 years = Rs. (40 x 2) = Rs. 80.
Principal = Rs. (215 - 80) = Rs. 135.
Hence, option (a) is correct.

50 (d)
In alphabet series generally, groups are of 3, 4, 5 and 6 letters depending upon the total number of letters.
Here, there are a total 16 letters. So a group of 4 letters is most probable.
a _ c d/ b _ d a/ c _a b/ d _ b c
By looking at the series we can say that all letters, i.e. a, b, c and d are written in ''clockwise alphabetical cyclic
order".
15 www.visionias.in ©Vision IAS

Telegram: UPSC_SARTHI
Hence, the arrangement will be:
a b c d / b c d a/ c d a b/ d a b c
∴ The letters that appear in the gaps are bcda.
Hence, option (d) is correct.

51 (b)
Option (a) is incorrect as the passage states that galaxies in the early universe were often flat and elongated, like
surfboards and pool noodles, and were rarely round, like volleyballs or frisbees.
Option (b) is correct because this statement accurately reflects the key observation mentioned in the passage -
most distant galaxies in the early universe exhibited flat and elongated shapes, resembling surfboards and pool
noodles. The lines “Researchers analyzing images from NASA’s James Webb Space Telescope have found that
galaxies in the early universe are often flat and elongated, like surfboards and pool noodles – and are rarely
round, like volleyballs or frisbees” clearly validate the claim in this answer option.
Option (c) is incorrect as the passage mentions that "volleyballs," or sphere-shaped galaxies, are the least
frequently identified type, not the most frequently identified ones - “The “volleyballs,” or sphere-shaped galaxies,
appear the most compact type on the cosmic “ocean” and were also the least frequently identified.”
Option (d) is incorrect as the passage notes that frisbee-shaped galaxies are as large as surfboard- and pool
noodle-shaped galaxies along the "horizon," and become more common closer to the "shore" in the nearby
universe. So, frisbee-shaped galaxies are commonly found along the ‘shore’, not along the ‘horizons.’

52 (b)
Option (a) is incorrect as the passage explicitly states that the nominal goal of self-reliance is not to protect
specific firms or sectors, but to build domestic resilience in a less secure world.
Option (b) is correct because the passage clearly expresses concern that while self-reliance may seem like a
reasonable response to a less secure world, it risks fuelling even greater systemic instability. It is duly captured in
the lines “Even if self-reliance is an understandable response to a world that is moving away from economic
openness, it risks fuelling even greater systemic instability.” Therefore, this answer option is incorrect.
Option (c) is incorrect because the passage suggests that the intensifying US-China rivalry has impaired
America's ability to keep the global market economy open. This is clearly a negative impact (contrary to what this
answer option states).
Option (d) is incorrect as the passage discusses the theory that a trusted and committed hegemon is necessary for
keeping international markets open. It does not suggest the need for multiple global hegemons which is
contradictory to the claim made in the lines “According to one theory of international relations, a trusted and
committed hegemon that enforces global rules and provides global public goods is a prerequisite to keeping
international markets open.”

53 (c)
Assumption 1 is incorrect. The passage does talk about the role of hegemons in keeping international markets
open. How resorting to protectionism poses a risk to open markets is also touched upon in the last lines of the
passage. However, there is absolutely no reference to erstwhile USSR. Therefore, this answer option is beyond the
scope of the passage.
Assumption 2 is incorrect because the passage suggests just the opposite. It states that today's autarkic tendencies
are a symptom of the fading Pax Americana. The intensifying US-China rivalry and the divide between
democratic and authoritarian regimes have impaired America's ability to keep the global market economy open.

54 (a)
Statement 1 is correct: The passage links individual contributions to society with a well-developed conscience
and aligned thoughts and actions, called integrity. The passage cites examples of Mahatma Gandhi and Mother
Teresa in the lines, “…many famous people all over the world have contributed to the welfare of society. All of
16 www.visionias.in ©Vision IAS

Telegram: UPSC_SARTHI
them are known for their integrity”. Therefore, it's reasonable to assume that individuals driven by strong integrity
are more likely to make significant and positive impacts on society. Hence, statement 1 is correct.
Statement 2 is not correct: Though the passage quotes the example of Mahatma Gandhi and Mother Teresa as
individuals having higher levels of consciousness, it does not mention that a super-pious conscience is a
prerequisite for an individual to be considered a great soul. Also, super-pious conscience may be a necessary, but
not a sufficient condition to be considered great souls. Hence, it would be incorrect to say that people with super -
pious conscience are always considered great souls.

55 (b)
N has 6 factors. Now, we can get 6 either by 6×1 or by 3×2.
So, if p and q are prime numbers, then N = p5 or p2q

Now, between 250 and 350 there’s no number of the form p5, but a number of the form p2q is possible.
If p = 2, then q can have 5 values (67,71,73,79,83)

If p = 3, then q can have 3 values (29,31,37)


If p = 5 then q can have 2 values (11,13)
So, total number of possible values = 10
Hence, option (b) is correct.

56 (a)
Total bill = Rs. 500
Total discount on cash payment = 30% of 500 = Rs. 150
∴ Total payment made by cash = 500 – 150 = Rs. 350
Total discount on card payment = 20% of 500 = Rs. 100
∴ Total payment made via card = 500 – 100 = Rs. 400
But he also gets 10% cashback on card payment.
∴ Total bill after 10% cashback = 400 – 10% of 400 = 400 – 40 = Rs. 360
Here, we can see that, cash payment is more profitable than card payment.
So, we can say that he paid Rs. 350 to get maximum profit.
Hence, option (a) is the correct answer.

57 (d)
We know that LCM is completely divisible by HCF. So, HCF/LCM should be of the form 1/x.
Therefore, HCF/LCM cannot be 3:35.
Hence, option (d) is the correct answer.

58 (b)
X is older than Y, Z is younger than W and V is as old as Y.
So, X > Y = V ; W > Z
Taking only first statement into consideration:

17 www.visionias.in ©Vision IAS

Telegram: UPSC_SARTHI
Nothing absolute can be said about Z and X
Now taking only second statement into consideration:
W≤V
So, X > Y = V ≥ W > Z
We can conclude that Z is younger than X.
Hence, option (b) is the correct answer.

59 (c)
Cyclicity of 2 and 8:
21 = 2 81 = 8
22 = 4 82 = 64
23 = 8 83 = 512
24 = 16 84 = 4096
The unit digits we will get:
8 + 2 = 10
4 + 64 = 68
8 + 512 = 520
16 + 4096 = 4112
So, we will get 0 as units digit one-half of the times.
There are 100 possible values of n from 1 to 100. So, one-half of these, i.e. 50, will produce a unit digit of 0 in the
expression 2n + 8n.
Hence, option (c) is the correct answer.

60 (d)
Total strength of school = 600
∵ Girls are 40% more than boys.
∴ Girls/Boys = 140/100 = 7/5
or Girls : Boys = 7 : 5
∴ Number of girls = (600/12) × 7 = 350 girls
Number of boys = (600/12) × 5 = 250 girls
Average weight of girls is 50 kg. Let average weight of boys and all students be x kg.
Now, according to the question,
⇒ 600x = 350 × 50 + 250x
⇒ 350x = 350 × 50
⇒ x = 50
Hence, average weight of all the students is 50 kg.
Alternate Method:
The average weight of girls is 50 kg and average weight of all students and boys is same. If the average weight of
all boys and the whole student community is the same, then the average weight of the girls must be the same too,
i.e. 50 kg will be the average weight of the whole student body.
Hence, option (d) is the correct answer.

18 www.visionias.in ©Vision IAS

Telegram: UPSC_SARTHI
61 (c)
Statement 1 is correct: People cannot fulfil the reason they live together unless they are well-organized and
adhere to a set of behavioural norms. The passage says, “The purpose for which people live together cannot be
realized unless they are properly organized and accept certain rules of conduct”. It can be inferred that to achieve
the final objective of people living together, they should be organized. Hence, statement 1 is correct.
Statement 2 is correct: The people who live in a given territory have a common goal that is centred on the
government. The passage says, “The government is also the focus of the common purpose of the people occupying
the definite territory. It is through this medium that common policies are determined, common affairs regulated,
and common interests promoted”. It can be inferred that joint goals are achieved directly or indirectly through the
government. Hence, statement 2 is correct.

62 (b)
Option (a) is not correct: The statement focuses on only one aspect of history. This option only covers a narrow
context discussed in the passage which is the relation of land with political history. However, the passage
highlights the impact of land on a wide aspect of human lives which is missing in the given answer option. Hence,
option (a) does not comprehensively capture the crux of the passage.
Option (b) is correct: The paragraph emphasizes how land is a crucial and integral component that has been
woven throughout human existence. It is the cornerstone of our economy, societies, and our very existence.
For example:
Land as a basic resource: We rely on land for food, fuel, clothing, and shelter. It's the foundation of our sustenance
and well-being.
Diverse roles: Land serves various purposes, from agriculture and mining to providing living space and
recreational areas.
Economic importance: Land is a crucial factor of production in various sectors and a source of wealth and power.
It can be assumed that land is a crucial component of human civilization and affects multiple and diverse aspects
of human lives. Hence, option (b) is correct.
Option (c) is not correct: The passage mentions the diverse forms in which land exists. It says, “It has many
physical forms like mountains, hills, plains, lowlands and valleys”. It is also mentioned that land is identified
closely with the natural environment. With respect to land usage and utility, the author only says that the land
supports many kinds of vegetation. This clearly indicates difference in land use patterns. Therefore, it would be
incorrect to say that land use pattern remains the same across the different physical forms. Hence, option (c) is
not correct.
Option (d) is not correct: The passage mentions land as an important determinant of economic process and
prosperity. But to build a necessary relation that without economic ownership of land there would be no prosperity
would not be correct as the passage does not suggest so. Hence, option (d) is not correct.

63 (c)
Let the original speed of car be s and time taken be t.
In first case, due to the problem, Rakesh covered the remaining distance at (4s/5) speed. So, time taken will be
5t/4.
This will lead to an increase in time taken by t/4.
Time difference = 50 – 41 = 9 minutes
t/4 = 9 minutes
or t = 36 minutes.
Difference in distance = 35 - 23 = 12 km
Hence, Rakesh travelled 12 km in 36 minutes.
So, Speed of Rakesh’s car = (12/36) × 60 = 20 km/hr.
Hence, option (c) is the correct answer.
19 www.visionias.in ©Vision IAS

Telegram: UPSC_SARTHI
64 (c)
1, 2, 3, …….4n
Out of these half must be even and half odd.
N12 + N22 will be even, when either both N1 and N2 are even or both are odd.
Number of favourable cases = 2nC2 + 2nC2
Total possible cases = 4nC2
So, Probability = (2 × 2nC2) / 4nC2 = 11/23
or (2n – 1) / (4n – 1) = 11/23
or n = 6
Hence, option (c) is the correct answer.

65 (d)
Considering first statement alone:
Nothing absolute can be said about the hats of Q and R, as Q might have black hat or white hat. Similarly, R might
have white or black hat. We cannot determine the colour of P’s hat.
Considering second statement alone:
Nothing concrete can be said about the hats of P and R as they can have either white or black hat.
Even considering both statements together, we are not able to deduce the colour of P’s hat.
Hence, option (d) is the correct answer.

66 (a)
-4 < 2x ≤ 2
Dividing by 2: -2 < x ≤ 1
It is equivalent to -2 < x and x ≤ 1.
Hence, option (a) is the correct answer.

67 (b)
Let total marks in the examination be x.
Now according to question,
20% of x + 120 = 40 % of x – 60
⇒ (x/5) – (2x/5) = - 180
⇒ x/5 = 180
⇒ x = 900
Hence, passing marks = 20% of 900 + 120 = 300 marks
Hence, option (b) is the correct answer.

20 www.visionias.in ©Vision IAS

Telegram: UPSC_SARTHI
68 (d)
All possibilities of scores are as follows:
Balls 1st 2nd 3rd 4th 5th 6th Total
Ways
1 0 1 1 6 6 6 20
2 0 1 1 4 4 4 14
3 0 1 1 4 4 6 16
4 0 1 1 4 6 6 18
Hence, 12 cannot be the score of the batsman after the over.
Hence, option (d) is the correct answer.

69 (c)
By looking at the given figure we can deduce that letter “d” represents all the applicants who are computer literate
and have knowledge of office rules and regulations, but do not have a bachelor’s degree.
Hence, option (c) is the correct answer.

70 (c)
Let total time taken to complete marathon = X minutes.
Now, according to the question,
(60/100) × X = 30 minutes
or X = [(30/60) × 100]
or X = 50 minutes
Hence, the runner has to run 50 – 30 = 20 minutes more in this marathon.
Hence, option (c) is the correct answer.

71 (b)
Option (a) is incorrect. The passage talks about the issues faced by children on account of unavailability of
electricity. However, there is no indication towards privatisation of electricity distribution. It is an extremely far-
fetched proposition. Hence, the given option is not correct as per the passage.
Option (b) is correct. The given option best reflects the logical inference, because the passage focuses on the
importance of energy for children's education which is not available and not accessible to many as seen in the
lines “They are often forced to seek other sources outside their houses, sometimes at gas stations or under
streetlamps, … poses serious health risks and oftentimes cannot be found locally.” In addition to academics, the
health of children is also affected. Therefore, it would be correct to say that the overall development of children is
marred.
Option (c) is incorrect. The context of building renewable sources of energy is not mentioned in the passage.
Hence, this option is beyond the scope of the passage and is not correct.
Option (d) is incorrect because option (a) is incorrect.

72 (b)
Option (a) is incorrect. The line “…human-wildlife coexistence is strongly linked and important to sustainable
development activities” highlights the importance of both human and wildlife for sustainable development. Their
coexistence is important, not their conflict. Therefore, this answer option is incorrect.
Option (b) is correct. The given option is the best crux of the passage because of the lines “With human-wildlife
conflict … and important to sustainable development activities. If not effectively managed, human-wildlife
21 www.visionias.in ©Vision IAS

Telegram: UPSC_SARTHI
conflict has the potential to negatively affect these activities and conservation much more broadly.” These lines
validate the assertion that reduction of human-wildlife conflicts is necessary for success of sustainable
development activities. Hence, this option best captures the crux of the passage.
Option (c) is incorrect. The given option could be a rational solution but not the crux. It just proposes a plausible
solution to reduce human-wildlife conflicts. Also, the context of decreasing population growth is not a part of the
passage. Hence, this answer option is incorrect.
Option (d) is incorrect. The line “Along with other threats, human-wildlife conflict has driven the decline of
once-abundant species and is pushing others to the brink of extinction”, only highlights the impact on animals.
The impact on humans, if any, is not a part of the passage. Also, to say that they are “equally affected” would be
arbitrary and hence wrong, at least based on the information at our disposal in the passage. Hence, this option is
not the best crux of the passage.

73 (c)
Option (a) is incorrect. The context that companies need AI for the development and distribution of games is not
a part of the passage. The line “This rise in popularity has resulted in a massive shift in the way video games are
developed, produced, and distributed” only specifies that there has been a significant change after the popularity.
Hence, this option is beyond the scope of the passage and is not correct.
Option (b) is incorrect. The given statement is true based on the line “Companies can’t expect to replicate their
success in Western markets with a “rinse and repeat” of existing strategies for new geographies.” However, this
is just a peripheral argument that lends support to the main theme of the passage - the globalization of games
based on localization. Hence, this answer option is incorrect.
Option (c) is correct. The lines “Different aspects of the game must be tailored for local playing conditions …
Companies that want to access (and profit from) the fast-growing gaming consumer base need to be prepared to
provide instruction, support, and moderation teams in multiple local languages” reflect the assertion given in the
option statement. These lines capture the essence of the passage – localisation is important to succeed in a
globalised gaming market. Hence, this is the best crux of the passage.
Option (d) is incorrect. The passage does not compare whether gaming companies are more successful in
Western markets or new geographies. The line “Companies can’t expect to replicate their success in Western
markets with a “rinse and repeat” of existing strategies for new geographies” merely mentions that companies
need to change their strategy in new geographies. Hence, this is not the correct crux as per the passage.

74 (d)
The name of 6 employees are: Ajay, Bijay, Chetan, David, Eshaan, and Firoz.
We must use statements 1, 2 and 3, as Bijay has been mentioned only in statement 1, Chetan has been mentioned
only in statement 2, and Eshaan has been mentioned only in statement 3.
Now, we need to account for Ajay, David and Firoz. We can account for Ajay using statement 1, and for David
and Firoz using statement 4.
Using statements 1 and 3, we get:
Ajay > Bijay > Eshaan
Using statements 2 and 4, we get:
Chetan = David > Firoz
However, we still need statement 5 to find out the relative salary of these two groups. Using statement 5, we get:
Chetan = David > Firoz > Ajay > Bijay > Eshaan
So, Chetan and David are the highest earners and to find this out we need to use all the given five statements.
Hence, option (d) is correct.

22 www.visionias.in ©Vision IAS

Telegram: UPSC_SARTHI
75 (c)
It is clear from the facts mentioned in the question that cost price of pure item X per kg = Rs. 250
Selling price of the mixture (item X with item Y) = Rs. 250
Profit is gained by mixing Y (which is free of cost) in X and Profit percentage = 30%
So, Cost price of the mixture (item X mixed with item Y) = 250/{1 + (30/100)} = Rs. 2500/13
By the rule of alligation, we have:

Ratio of item Y to item X in the mixture = (750/13) : (2500/13) = 3 : 10


Hence, option (c) is the correct answer.

76 (c)
The ratio of literate males to literate females is 6 : 7. Let their number be 6x and 7x respectively.
It is also given that 6x is equal to 2400. So, 7x will be 2800.
The ratio of the number of illiterate males to illiterate females is 3:4. Let the number of illiterate males and
illiterate females be 3y and 4y respectively.
Total number of males = illiterate males + literate males = 3y + 2400
Total number of females = illiterate females + literate females = 4y + 2800
Now, according to the question,
(2400 + 3y)/(2800 + 4y) = 4/5
Or 12000 + 15y = 11200y + 16y
Or y = 12000 - 11200
Or y = 800
So, the difference between the number of males and females in the village = (4y + 2800) – (3y + 2400) = y + 400
= 800 + 400 = 1200
Hence, option (c) is correct.

77 (b)
(a) These are 4 numbers. Sum of 13 + 05 + 35 + 11 = 64 = 43
(b) These are 3 numbers. Sum of 02 + 09 + 17 = 28, which is not equal to 33 = 27
(c) These are 5 numbers. Sum of 34 + 29 + 41 + 07 + 14 = 125 = 53

23 www.visionias.in ©Vision IAS

Telegram: UPSC_SARTHI
(d) These are 6 numbers. Sum of 76 + 24 + 16 + 64 + 04 + 32 = 216 = 63
Thus, the group 02, 09, 17 is different from the others.
Hence, option (b) is correct.

78 (b)
Here, we are just summing up the digits of the numbers till we get a single digit answer.
Assume =+

12 3 45 = (1+2) + 3 + (4+5) = 15 → 1 + 5 = 6,
67 31 56 = (6+7) + (3+1) + (5+6) = 28 → 2 + 8 = 10; 1 + 0 = 1,
54 23 89 = (5+4) + (2+3) + (8+9) = 31 → 3 + 1 = 4
73 34 21 = (7+3) + (3+4) + (2+1) = 20 → 2 + 0 = 2
Hence, option (b) is correct.

79 (a)
We can draw the following possible Venn diagram based on the given three statements:

It is clear that, Conclusion-I (All pen are sharpners) is always true.


But, Conclusion-II (All sharpners are pencils) is not always true.
Therefore, option (a) is the correct answer.

80 (a)
Let the angles of the triangle be 2x, 3x and 4x respectively.
So, 2x + 3x + 4x = 9x = 180
Or x = 20.
The angles of the triangle are 40˚, 60˚ and 80˚ respectively.
As all the angles of the triangle are of different measures, all the sides of the triangle must be of different lengths.
Hence, option (a) is the correct answer.

Copyright © by Vision IAS


All rights are reserved. No part of this document may be reproduced, stored in a retrieval system or
transmitted in any form or by any means, electronic, mechanical, photocopying, recording or otherwise,
without prior permission of Vision IAS.

24 www.visionias.in ©Vision IAS

Telegram: UPSC_SARTHI

You might also like